Q13

 
vincent.m
Thanks Received: 0
Jackie Chiles
Jackie Chiles
 
Posts: 30
Joined: September 08th, 2013
 
 
 

Q13

by vincent.m Mon Jun 02, 2014 10:03 pm

Hi,

I just need help with this game, especially the diagram. I realize there is already one made, but can someone explain it a little more. Thank you!
User avatar
 
maryadkins
Thanks Received: 640
Atticus Finch
Atticus Finch
 
Posts: 1261
Joined: March 23rd, 2011
 
 
 

Re: Q13

by maryadkins Sun Jun 08, 2014 10:25 am

It's a really tricky one...I actually wasn't able to make the inferences up front that Matt made, either. But let's figure out what you can do.

So we have the 3 groups of people, and the offices 101-107.

Focus on stacking up the most restrictive rules to figure out where you have the most restrictions...as I read through the first 3 rules, I start to see that G is really limited.

G cannot be next to: V, W, or R, and it can't be in 101 or 107 (neither can F).

That's pretty restrictive! So what does that mean? I now want to turn that negative stack of rules into a positive inference. G can ONLY be next to: F, Q and S, and it has to be somewhere between 102 and 206"”that's 5 slots.

We don't know what this means yet, but we want to write it down.

Finally, we know that this rule that if W and V are next to each other, that means R has to be next to V, too. So that would mean W is on one side of V and R is on the other side: RVW, or WVR. But this is a conditional, so we don't know how it's going to play out yet. The trigger is if W ends up next to V"”that will mean R will also have to be next to V. Let's just keep an eye on it.

From this point, I would do questions 11 and 13 first, hoping to come up with more inferences, because they're conditionals and give us new information to work with...

Let me know if you still want help working through this set up.
 
AnnaT620
Thanks Received: 0
Elle Woods
Elle Woods
 
Posts: 51
Joined: May 25th, 2020
 
 
 

Re: Q13

by AnnaT620 Tue Feb 23, 2021 10:12 am

Hello Team

Please can you give some more details on how to solve these questions? I also didn't see the inference that F and G must either be consecutive or separated by exactly one office. How would you approach question 13 - by testing each answer choice?

Thank you!
Anna
 
Laura Damone
Thanks Received: 94
Atticus Finch
Atticus Finch
 
Posts: 468
Joined: February 17th, 2011
 
 
 

Re: Q13

by Laura Damone Thu Feb 25, 2021 8:16 pm

Hi Anna!

#13 is a conditional question, so I'd build R in position 4 into a new diagram. I personally do all my new hypothetical diagrams underneath the master, in a little chart.

__ __ __ R __ __ __

So, what does this tell me? Well, the R/G antichunk means G can't go in 3 or 5. And G is already excluded from 1 and 7. So that leaves only 2 and 6 as possibilities for G. Since I don't see any other inferences, I used question-specific framing and drew out both possibilities:

__ G __ R __ __ __

__ __ __ R __ G __

Once G is placed, we can create exclusion inferences for V and W under the slots adjacent to G. Looking first at the bottom frame, this would exclude V and W from 5 and 7, thereby forcing them both onto the 1-2-3 side of the diagram. Now, ask yourself "who does that impact?" Well, if two of those three slots must be filled by V and W, there's no way put F over there without violating rule 1. And rule 2 tells us that F isn't in slot 7, so that leaves only slot 5 for F.

I can look at the other frame and infer that it's just a mirror image of the first frame, meaning F is the element that's stuck in 103 or 105.

My final frame, with parentheses representing clouds, looks like so:

(V, W, Q/S) R F G S/Q

If I didn't see that the second frame was a mirror image of the first and drew it out, it would look like so:

Q/S G F R ( V, W, S/Q )

By using question specific frames, I avoid testing answers. That said, there's never a question you can't plug and chug. To test answers on this question, because it's a must be true, you'd test an alternative position for the element in question. So, to test (A), you'd put F somewhere other than 103 or 105. If you can make it work, the answer is wrong. If you can't make it work, the answer is correct.

Hope this helps1
Laura Damone
LSAT Content & Curriculum Lead | Manhattan Prep